LSAT and Law School Admissions Forum

Get expert LSAT preparation and law school admissions advice from PowerScore Test Preparation.

 GLMDYP
  • Posts: 100
  • Joined: Aug 19, 2013
|
#10420
Hi Powerscore!
For this question, I don't understand why if Quore don't attain 20 percent productivity increase, it must go bankrupt. The stimulus simply said that if 10 percent productivity increase is not attained, Quore will go bankrupt. It only said that 20 percent is attainable, but not a necessity for avoiding bankrupt.
Thanks!
 David Boyle
PowerScore Staff
  • PowerScore Staff
  • Posts: 836
  • Joined: Jun 07, 2013
|
#10470
GLMDYP wrote:Hi Powerscore!
For this question, I don't understand why if Quore don't attain 20 percent productivity increase, it must go bankrupt. The stimulus simply said that if 10 percent productivity increase is not attained, Quore will go bankrupt. It only said that 20 percent is attainable, but not a necessity for avoiding bankrupt.
Thanks!
Hello GLMDYP,

Because "if a 10 percent productivity increase is possible, then a 20 percent increase is attainable". And Quore needs the 10 percent increase to be possible (which is not the exact same as "attainable"); so if that is not possible, then...big trouble for Quore!

David
 melissalsatprep
  • Posts: 1
  • Joined: Aug 27, 2014
|
#16212
I am working through the Type Training book to try to improve my diagramming of conditional reasoning statements. I am having a heck of a time understanding why the correct diagram to the first sentence of the stimulus.

I know that it should be: ~up 10% --> B
But when I initially read it, I saw "must" as a NC indicator, thus "must increase productivity by 10%", lead me to diagram: B --> ~up 10%

Can anyone help me unravel the first sentence? I am not the best at "seeing" the diagram, thus I heavily rely on the NC/SC indicators. Any tips on how to recognize when the indicator is there, but it is an exception?
Thanks,
Melissa
 Nikki Siclunov
PowerScore Staff
  • PowerScore Staff
  • Posts: 1362
  • Joined: Aug 02, 2011
|
#16217
Hi Melissa,

You are absolutely correct, "must" is a NC indicator, and this is not an exception. Let's take a look at the sentence in question:
Quore must increase productivity 10% [...], or it will go bankrupt.
Quore has two options: either increase productivity by 10%, or go bankrupt. So, if Quore does not increase productivity, it will bankrupt, and - by the contrapositive - if Quore does not go bankrupt, it must increase productivity by 10%:

~ increase 10% :arrow: bankrupt
~ bankrupt :arrow: increase 10%

As you can see from the contrapositive, the 10% increase is a necessary condition ("must") for Quore to avoid bankruptcy.

Generally speaking, the construction "either A or B must occur" can be represented as:

~A :arrow: B
~B :arrow: A

Note that it is still possible for A and B to occur at the same time: Quore can increase productivity and still go bankrupt.

Hope this helps!
 LSAT2020
  • Posts: 31
  • Joined: Jun 24, 2020
|
#77305
I am really confused on why the answer choice is E. I chose B. Is B wrong because it is a mistaken negation? The reason I am struggling to wrap my head around answer choice E is because of the assertion surrounding the 20%. When I was diagramming out the stimulus, I had the following:

Avoid bankruptcy :arrow: 10% increase over the course of the next two years

/10% increase over the course of the next two years :arrow: /avoid bankruptcy

So basically, all we can say is true is that Quore has to increase by a minimum of 10% over a period of two years.

The last sentence in stimulus threw me off. Where was the author headed by pointing this out? Are we expected to interpret the sentence at face value, and thus conclude that Quote will in fact increase by 20% over the course of the next two years?
Even if we do take it at face value, it doesn't change the fact that the minimum increase that Quore has to have in order to avoid bankruptcy is 10%. If it does increase by 20% as the author is telling us, then that's even better. However, the way I am interpreting this stimulus, it doesn't imply that Quore MUST get to that 20% in order to avoid bankruptcy. I don't see anything that indicates that now the bar to be reached is 20% rather than 10%.

I'm clearly missing something here and would greatly appreciate clarification. Thanks in advance!
User avatar
 KelseyWoods
PowerScore Staff
  • PowerScore Staff
  • Posts: 1079
  • Joined: Jun 26, 2013
|
#80209
Hi LSAT2020!

Your diagram of the first sentence is correct:

Bankrupt :arrow: 10% productivity increase

10% productivity increase :arrow: Bankrupt

That last sentence is just another conditional statement. Diagram it as so:

10% productivity increase possible :arrow: 20% increase attainable

You can then take the contrapositive so that you can link it to your conditional statement in the previous sentence. If a 10% productivity increase is not possible, that's the same as saying they're not going to attain a 10% productivity increase. So:

20% increase attainable :arrow: 10% productivity increase possible :arrow: Bankrupt

That gives us answer choice (E): 20% increase attainable :arrow: Bankrupt

Hope this helps!

Best,
Kelsey
 SammyWu11201
  • Posts: 29
  • Joined: Jun 29, 2020
|
#81635
Is B wrong because the stimulus says you MUST have one of the two: increase productivity by 10% or go bankrupt? Therefore, there are no situations in which you don't have at least one of the two options. Maybe I'm mistaken, but after reading the LR Bible, I have the idea that "either A or B" can sometimes be diagrammed as A negative double arrow B, meaning there is a possibility for both of them to not be present. So what's the difference between an "either...or" statement that has the negative in the sufficient condition and one that has it in the negative condition?
 Adam Tyson
PowerScore Staff
  • PowerScore Staff
  • Posts: 5153
  • Joined: Apr 14, 2011
|
#81658
Your analysis of the effect of "or" in the stimulus is correct, SammyWu11201 - the company must either get that increase or go bankrupt, or both (they might get the increase and go bankrupt anyway). Conditionally you can think of that as "if you don't have one of these things, then you will have the other."

The problem with answer B is not the either/or nature of the relationship, but that the answer is a Mistaken Negation. The stimulus proves this relationship:

10% Increase :arrow: Bankrupt

Answer B gives us this:

10% Increase :arrow: Bankrupt

Whoops! You cannot just change the positive/negative aspects of the terms and leave them in the original order!

You could also call this a Mistaken Reversal, if you saw the original relationship as:

Bankrupt :arrow: 10% Increase

Then answer B looks like the author reversed the order but left the positive/negative aspects the same, which is also a no-no.

Your last question was:
So what's the difference between an "either...or" statement that has the negative in the sufficient condition and one that has it in the negative condition?
The answer is that in the first case, you must have at least one of the two conditions and could have both, while in the latter you could have one or the other OR NEITHER, but you cannot have both.

Get the most out of your LSAT Prep Plus subscription.

Analyze and track your performance with our Testing and Analytics Package.